Why is B wrong?
Why is B wrong?
Raheel on April 19 at 09:40PM
  • December 2001 LSAT
  • SEC3
  • Q2
1
Reply
Why not A
Why not A? Thank you!
DevinFuller on October 12 at 10:50PM
  • December 2001 LSAT
  • SEC3
  • Q16
1
Reply
A?
Why is it not A?
burdal on March 23, 2023
  • December 2001 LSAT
  • SEC3
  • Q7
1
Reply
why is A wrong?
This is how I diagrammed the argument's conclusion: Superior mastery of athletic technique --> ch...
Abigail-Okereke on October 5, 2022
  • December 2001 LSAT
  • SEC3
  • Q3
1
Reply
A vs. C
Can someone break down the difference between these two choices?
annaj on August 6, 2022
  • December 2001 LSAT
  • SEC3
  • Q12
1
Reply
Why is A correct?
Can you explain why A is correct and B is wrong? Thanks.
capoleway@gmail.com on September 20, 2021
  • December 2001 LSAT
  • SEC3
  • Q12
2
Replies
Why A?
I picked D, can someone explain why A is the correct answer? Thanks!
liwenong28 on January 29, 2021
  • December 2001 LSAT
  • SEC3
  • Q13
1
Reply
??
Why not answer choice C? If the money pot is bigger that doesn't necessarily mean this is the bes...
Anthony-Resendes on January 7, 2021
  • December 2001 LSAT
  • SEC3
  • Q21
1
Reply
Question Explanation
Please could you explain why C is not the answer here? It seemed less difficult to prove than D. ...
Anna20 on August 7, 2020
  • December 2001 LSAT
  • SEC3
  • Q17
3
Replies
Why is E incorrect?
Why is E incorrect?
Shiyi-Zhang on May 4, 2019
  • December 2001 LSAT
  • SEC3
  • Q18
1
Reply
Answer D
Why can't answer choice D be correct? Thanks
rolltribe on September 11, 2015
  • December 2001 LSAT
  • SEC3
  • Q20
1
Reply
Mildly drought-stressed
How can you infer from the conclusion that "watering plants just enough" is equal to "mildly drou...
rolltribe on September 9, 2015
  • December 2001 LSAT
  • SEC3
  • Q11
1
Reply